Inspired by Issac Newton

Algebra Level 3

k = 1 20 ( α 3 k + β 3 k + ξ 3 k ) \large \sum _{ k=1 }^{ 20 }{ \left( { \alpha }^{ 3k }+{ \beta }^{ 3k }+{ \xi }^{ 3k } \right) }

{ f ( 1 ) = 9 f ( 1 ) = 7 f ( 3 ) = 19 \begin{cases} {f\left( -1 \right) =-9} \\ {f\left( 1 \right) =-7 } \\ {f\left( 3 \right) =19} \end{cases}

If f ( x ) f\left( x \right) is monic cubic polynomial having roots α , β , ξ \alpha ,\beta ,\xi . Then evaluate topmost expression modulo 17.


The answer is 15.

This section requires Javascript.
You are seeing this because something didn't load right. We suggest you, (a) try refreshing the page, (b) enabling javascript if it is disabled on your browser and, finally, (c) loading the non-javascript version of this page . We're sorry about the hassle.

3 solutions

Laurent Shorts
Feb 15, 2017

Finding f ( x ) f(x) :

If f ( x ) = ( x + 1 ) ( x 1 ) ( x 3 ) + a ( x 1 ) ( x 3 ) + b ( x 3 ) + c f(x)=(x+1)(x-1)(x-3)+a(x-1)(x-3)+b(x-3)+c , then: { 8 a 4 b + c = 9 2 b + c = 7 c = 19 { a = 3 b = 13 c = 19 \begin{cases}8a\!\!\!\!&-4b\!\!\!\!&+c\!\!\!\!&=-9\\&-2b\!\!\!\!&+c\!\!\!\!&=-7\\&&\ \ \ c\!\!\!\!&=19\end{cases}\ \Longleftrightarrow\ \begin{cases}a=3\\b=13\\c=19\end{cases} f ( x ) = ( ( x + 1 + a ) ( x 1 ) + b ) ( x 3 ) + c = x 3 8 \Longrightarrow \ f(x)=((x+1+a)(x-1)+b)(x-3)+c=\boxed{x^3-8}

Finding the value of the expression:

Clearly, α 3 = β 3 = ξ 3 = 8 \alpha^3=\beta^3=\xi^3=8 and α 3 k + β 3 k + ξ 3 k = 8 k + 8 k + 8 k = 3 8 k \alpha^{3k}+\beta^{3k}+\xi^{3k}=8^k+8^k+8^k=3·8^k .

The topmost expression is k = 1 20 3 8 k = 3 k = 1 20 8 k = 3 8 8 20 1 8 1 = 24 7 ( 8 20 1 ) = 24 7 ( 2 60 1 ) \sum_{k=1}^{20}{3·8^k}=3·\sum_{k=1}^{20}{8^k}=3·8·\dfrac{8^{20}-1}{8-1}=\frac{24}{7}(8^{20}-1)=\frac{24}{7}(2^{60}-1) .

Modulo 17, as 2 4 = 16 1 2^4=16\equiv -1 : the answer is 24 7 ( 2 60 1 ) 7 7 ( ( 2 4 ) 15 1 ) ( 1 ) 15 1 = 2 15 \frac{24}{7}(2^{60}-1)\equiv \frac{7}{7}\left((2^4)^{15}-1\right)\equiv(-1)^{15}-1=-2\equiv\boxed{15} .

Shivamani Patil
Jul 3, 2015

Let f ( x ) = x 3 + b x 2 + c x + d f\left( x \right) ={ x }^{ 3 }+b{ x }^{ 2 }+cx+d

{ b + c + d = 8 b c + d = 8 27 + 8 b + 3 c + d = 19 \therefore \begin{cases} {b+c+d=-8} \\ {b-c+d=-8 } \\ {27+8b+3c+d=19} \end{cases}

b = c = 0 , d = 8 \Rightarrow b=c=0,d=-8

f ( x ) = x 3 8 \Rightarrow f\left( x \right) ={ x }^{ 3 }-8

Now roots of f ( x ) f\left( x \right) are 2 , 2 ω , 2 ω 2 2,2\omega ,{ 2\omega }^{ 2 } where ω \omega is cube root of unity.

k = 1 20 ( α 3 k + β 3 k + ξ 3 k ) = k = 1 20 ( 2 3 k + ( 2 ω ) 3 k + ( 2 ω 2 ) 3 k ) \therefore \sum _{ k=1 }^{ 20 }{ \left( { \alpha }^{ 3k }+{ \beta }^{ 3k }+{ \xi }^{ 3k } \right) } =\sum _{ k=1 }^{ 20 }{ \left( 2^{ 3k }+{ (2\omega ) }^{ 3k }+{ ({ 2\omega }^{ 2 }) }^{ 3k } \right) }

= k = 1 20 2 3 k ( 1 + ω 3 k + ω 6 k ) = k = 1 20 2 3 k ( 3 ) ω 3 = 1 =\sum _{ k=1 }^{ 20 }{ 2^{ 3k }\left( 1+{ \omega }^{ 3k }+{ \omega }^{ 6k } \right) } =\sum _{ k=1 }^{ 20 }{ 2^{ 3k }\left( 3 \right) } \because { \omega }^{ 3 }=1

k = 1 20 2 3 k ( 3 ) \sum _{ k=1 }^{ 20 }{ 2^{ 3k }\left( 3 \right) } \quad mod 17 17 = 15 =15

How did you get your last statement?

I mean, How can you directly say, k = 1 20 2 3 k ( 3 ) = 15 ( m o d 17 ) ? \sum _ {k=1}^{20} {2^{3k} \left (3 \right) } \quad =15(mod \quad 17)?

Mehul Arora - 5 years, 11 months ago

Log in to reply

Here's a nice shortcut I used: k = 1 16 8 k = 1 8 17 1 8 1 \sum _{ k=1 }^{ 16 }{ { 8 }^{ k } } =\frac { 1-{ 8 }^{ 17 } }{ 1-8 } -1 Use that 8 17 8 ( m o d 17 ) 8^{17} \equiv 8 \pmod{17} (Fermat's little theorem) so the fraction becomes 1 and the sum is 0 mod 17. Therefore, what you really need to find is: k = 1 4 8 k 5 ( m o d 17 ) \sum _{ k=1 }^{ 4 }{ { 8 }^{ k } } \equiv5\pmod{17} I did this last sum by hand, which isn't all that time consuming since it's only 4 terms. Then just multiply by 3 to get the answer.

Dylan Pentland - 5 years, 11 months ago

Log in to reply

I messed up at the last step. Forgot to multiply by 3. Dumbest mistake in the history of brilliant perhaps.

Kunal Verma - 5 years, 7 months ago

Last statement is computational work and can be solved .Crux of problem was to get it to last step.And observe it is gp.

shivamani patil - 5 years, 11 months ago

I got till as far as the roots of the Polynomial. How to proceed Further?

Please elaborate a bit on your solution :)

Mehul Arora - 5 years, 11 months ago

Log in to reply

Actually I took 2^3k common out of whole expression.

shivamani patil - 5 years, 11 months ago
Chinmay Kharche
Jan 18, 2020

Skills dsv sdvdfbfgnhnghmb hmghm m ghghvgc cgnfgc nfgn fdb fdnfvg ndf nf nfgb mfbh fcgm b

0 pending reports

×

Problem Loading...

Note Loading...

Set Loading...